regular language equality prove by induction












2












$begingroup$


let $Lsubseteq{0,1}^*$ be declared by the following conditions:



a. $0, 01in L$.



b. if $w_1,w_2in L$ so $w_1cdot w_2in L$.



c. if $wcdot 0in L$ so $w in L$.



prove that $L={w| w=epsilon: or : win 0cdot (0,1)^* wedge not : contains : 11: as : substring }$



conclude that $L$ is a regular language.



I tried to prove the first side of the equality using induction but I am stuck in the step of the induction. I tried to take a word of size $n+1$ but I am not sure how to devide the word.



I would like to get assist also with the second part of the equality that I am not sure how to prove.










share|cite|improve this question











$endgroup$

















    2












    $begingroup$


    let $Lsubseteq{0,1}^*$ be declared by the following conditions:



    a. $0, 01in L$.



    b. if $w_1,w_2in L$ so $w_1cdot w_2in L$.



    c. if $wcdot 0in L$ so $w in L$.



    prove that $L={w| w=epsilon: or : win 0cdot (0,1)^* wedge not : contains : 11: as : substring }$



    conclude that $L$ is a regular language.



    I tried to prove the first side of the equality using induction but I am stuck in the step of the induction. I tried to take a word of size $n+1$ but I am not sure how to devide the word.



    I would like to get assist also with the second part of the equality that I am not sure how to prove.










    share|cite|improve this question











    $endgroup$















      2












      2








      2





      $begingroup$


      let $Lsubseteq{0,1}^*$ be declared by the following conditions:



      a. $0, 01in L$.



      b. if $w_1,w_2in L$ so $w_1cdot w_2in L$.



      c. if $wcdot 0in L$ so $w in L$.



      prove that $L={w| w=epsilon: or : win 0cdot (0,1)^* wedge not : contains : 11: as : substring }$



      conclude that $L$ is a regular language.



      I tried to prove the first side of the equality using induction but I am stuck in the step of the induction. I tried to take a word of size $n+1$ but I am not sure how to devide the word.



      I would like to get assist also with the second part of the equality that I am not sure how to prove.










      share|cite|improve this question











      $endgroup$




      let $Lsubseteq{0,1}^*$ be declared by the following conditions:



      a. $0, 01in L$.



      b. if $w_1,w_2in L$ so $w_1cdot w_2in L$.



      c. if $wcdot 0in L$ so $w in L$.



      prove that $L={w| w=epsilon: or : win 0cdot (0,1)^* wedge not : contains : 11: as : substring }$



      conclude that $L$ is a regular language.



      I tried to prove the first side of the equality using induction but I am stuck in the step of the induction. I tried to take a word of size $n+1$ but I am not sure how to devide the word.



      I would like to get assist also with the second part of the equality that I am not sure how to prove.







      induction computer-science regular-language






      share|cite|improve this question















      share|cite|improve this question













      share|cite|improve this question




      share|cite|improve this question








      edited Dec 9 '18 at 0:23









      Math1000

      19.1k31745




      19.1k31745










      asked Dec 8 '18 at 22:34









      UltimateMathUltimateMath

      896




      896






















          1 Answer
          1






          active

          oldest

          votes


















          2












          $begingroup$

          I think it is most straightforward to prove both inclusions by induction. Namely, let $M$ be the language described by the condition you have above; we wish to show that $L subseteq M$ and $M subseteq L$.



          For $L subseteq M$, we will actually use structural induction, rather than induction on the length of the string. What this means is that we will induct on the fact that the string is in $L$ because it satisfies some part of the definition above, i.e it is built from strings we already know are in $L$; we then show that, assuming these strings are in $M$, so too is the string constructed from them.



          It is clear that $0, 01 in M$. So we have our base cases, and proceed with the inductive cases.



          Let $w = w_1 cdot w_2$ where $w_1, w_2 in L$. By induction hypothesis, this means that $w_1, w_2 in M$. It follows that neither of them contains a $11$ and both begin with $0$. Then $w$ also begins with $0$, and it does not contain a $11$ -- the only possible place this could happen is from the last character of $w_1$ and the first one of $w_2$, but $w_2$ begins with $0$. Thus, $w in M$.



          Now let $w cdot 0 = w'$ where $w' in L$. By induction hypothesis, $w' in M$, so it begins with $0$; so too must $w$. Additionally, $w$ cannot contain a $11$, as this would mean $w'$ does as well. Thus, $w in M$. (Actually we have to be a little careful here to consider the case where $w = epsilon$ separately, but you can do this case explicitly).



          This proves the inclusion $L subseteq M$. For the other direction, we proceed by strong induction on the length of the string. Base cases should be $epsilon, 0$; the inclusion of both of these in $L$ is straightforward.



          Let $w in M$ have length at least $2$. It must end in a $0$ or a $1$. If it ends in a $0$, then let $w = w' cdot 0$. Since $w in M$, it must begin with a $0$ and not contain $11$; then $w'$ does as well, so $w' in M$. But by induction hypothesis, this means $w' in L$, and $w = w' cdot 0$, both of which are in $L$; then $w in L$.



          If $w$ ends in a $1$, then we know that it must end in $01$ (it has length at least $2$ and cannot end in $11$). The same argument as before, taking $w = w' cdot 01$ suffices (this is why we need strong induction, or at least must consider all strings of length up to $n-2$ as well).






          share|cite|improve this answer









          $endgroup$













          • $begingroup$
            It is the first time I uses structrual induction. can you tell me how the hypothesis gonna be?
            $endgroup$
            – UltimateMath
            Dec 10 '18 at 8:47













          Your Answer





          StackExchange.ifUsing("editor", function () {
          return StackExchange.using("mathjaxEditing", function () {
          StackExchange.MarkdownEditor.creationCallbacks.add(function (editor, postfix) {
          StackExchange.mathjaxEditing.prepareWmdForMathJax(editor, postfix, [["$", "$"], ["\\(","\\)"]]);
          });
          });
          }, "mathjax-editing");

          StackExchange.ready(function() {
          var channelOptions = {
          tags: "".split(" "),
          id: "69"
          };
          initTagRenderer("".split(" "), "".split(" "), channelOptions);

          StackExchange.using("externalEditor", function() {
          // Have to fire editor after snippets, if snippets enabled
          if (StackExchange.settings.snippets.snippetsEnabled) {
          StackExchange.using("snippets", function() {
          createEditor();
          });
          }
          else {
          createEditor();
          }
          });

          function createEditor() {
          StackExchange.prepareEditor({
          heartbeatType: 'answer',
          autoActivateHeartbeat: false,
          convertImagesToLinks: true,
          noModals: true,
          showLowRepImageUploadWarning: true,
          reputationToPostImages: 10,
          bindNavPrevention: true,
          postfix: "",
          imageUploader: {
          brandingHtml: "Powered by u003ca class="icon-imgur-white" href="https://imgur.com/"u003eu003c/au003e",
          contentPolicyHtml: "User contributions licensed under u003ca href="https://creativecommons.org/licenses/by-sa/3.0/"u003ecc by-sa 3.0 with attribution requiredu003c/au003e u003ca href="https://stackoverflow.com/legal/content-policy"u003e(content policy)u003c/au003e",
          allowUrls: true
          },
          noCode: true, onDemand: true,
          discardSelector: ".discard-answer"
          ,immediatelyShowMarkdownHelp:true
          });


          }
          });














          draft saved

          draft discarded


















          StackExchange.ready(
          function () {
          StackExchange.openid.initPostLogin('.new-post-login', 'https%3a%2f%2fmath.stackexchange.com%2fquestions%2f3031736%2fregular-language-equality-prove-by-induction%23new-answer', 'question_page');
          }
          );

          Post as a guest















          Required, but never shown

























          1 Answer
          1






          active

          oldest

          votes








          1 Answer
          1






          active

          oldest

          votes









          active

          oldest

          votes






          active

          oldest

          votes









          2












          $begingroup$

          I think it is most straightforward to prove both inclusions by induction. Namely, let $M$ be the language described by the condition you have above; we wish to show that $L subseteq M$ and $M subseteq L$.



          For $L subseteq M$, we will actually use structural induction, rather than induction on the length of the string. What this means is that we will induct on the fact that the string is in $L$ because it satisfies some part of the definition above, i.e it is built from strings we already know are in $L$; we then show that, assuming these strings are in $M$, so too is the string constructed from them.



          It is clear that $0, 01 in M$. So we have our base cases, and proceed with the inductive cases.



          Let $w = w_1 cdot w_2$ where $w_1, w_2 in L$. By induction hypothesis, this means that $w_1, w_2 in M$. It follows that neither of them contains a $11$ and both begin with $0$. Then $w$ also begins with $0$, and it does not contain a $11$ -- the only possible place this could happen is from the last character of $w_1$ and the first one of $w_2$, but $w_2$ begins with $0$. Thus, $w in M$.



          Now let $w cdot 0 = w'$ where $w' in L$. By induction hypothesis, $w' in M$, so it begins with $0$; so too must $w$. Additionally, $w$ cannot contain a $11$, as this would mean $w'$ does as well. Thus, $w in M$. (Actually we have to be a little careful here to consider the case where $w = epsilon$ separately, but you can do this case explicitly).



          This proves the inclusion $L subseteq M$. For the other direction, we proceed by strong induction on the length of the string. Base cases should be $epsilon, 0$; the inclusion of both of these in $L$ is straightforward.



          Let $w in M$ have length at least $2$. It must end in a $0$ or a $1$. If it ends in a $0$, then let $w = w' cdot 0$. Since $w in M$, it must begin with a $0$ and not contain $11$; then $w'$ does as well, so $w' in M$. But by induction hypothesis, this means $w' in L$, and $w = w' cdot 0$, both of which are in $L$; then $w in L$.



          If $w$ ends in a $1$, then we know that it must end in $01$ (it has length at least $2$ and cannot end in $11$). The same argument as before, taking $w = w' cdot 01$ suffices (this is why we need strong induction, or at least must consider all strings of length up to $n-2$ as well).






          share|cite|improve this answer









          $endgroup$













          • $begingroup$
            It is the first time I uses structrual induction. can you tell me how the hypothesis gonna be?
            $endgroup$
            – UltimateMath
            Dec 10 '18 at 8:47


















          2












          $begingroup$

          I think it is most straightforward to prove both inclusions by induction. Namely, let $M$ be the language described by the condition you have above; we wish to show that $L subseteq M$ and $M subseteq L$.



          For $L subseteq M$, we will actually use structural induction, rather than induction on the length of the string. What this means is that we will induct on the fact that the string is in $L$ because it satisfies some part of the definition above, i.e it is built from strings we already know are in $L$; we then show that, assuming these strings are in $M$, so too is the string constructed from them.



          It is clear that $0, 01 in M$. So we have our base cases, and proceed with the inductive cases.



          Let $w = w_1 cdot w_2$ where $w_1, w_2 in L$. By induction hypothesis, this means that $w_1, w_2 in M$. It follows that neither of them contains a $11$ and both begin with $0$. Then $w$ also begins with $0$, and it does not contain a $11$ -- the only possible place this could happen is from the last character of $w_1$ and the first one of $w_2$, but $w_2$ begins with $0$. Thus, $w in M$.



          Now let $w cdot 0 = w'$ where $w' in L$. By induction hypothesis, $w' in M$, so it begins with $0$; so too must $w$. Additionally, $w$ cannot contain a $11$, as this would mean $w'$ does as well. Thus, $w in M$. (Actually we have to be a little careful here to consider the case where $w = epsilon$ separately, but you can do this case explicitly).



          This proves the inclusion $L subseteq M$. For the other direction, we proceed by strong induction on the length of the string. Base cases should be $epsilon, 0$; the inclusion of both of these in $L$ is straightforward.



          Let $w in M$ have length at least $2$. It must end in a $0$ or a $1$. If it ends in a $0$, then let $w = w' cdot 0$. Since $w in M$, it must begin with a $0$ and not contain $11$; then $w'$ does as well, so $w' in M$. But by induction hypothesis, this means $w' in L$, and $w = w' cdot 0$, both of which are in $L$; then $w in L$.



          If $w$ ends in a $1$, then we know that it must end in $01$ (it has length at least $2$ and cannot end in $11$). The same argument as before, taking $w = w' cdot 01$ suffices (this is why we need strong induction, or at least must consider all strings of length up to $n-2$ as well).






          share|cite|improve this answer









          $endgroup$













          • $begingroup$
            It is the first time I uses structrual induction. can you tell me how the hypothesis gonna be?
            $endgroup$
            – UltimateMath
            Dec 10 '18 at 8:47
















          2












          2








          2





          $begingroup$

          I think it is most straightforward to prove both inclusions by induction. Namely, let $M$ be the language described by the condition you have above; we wish to show that $L subseteq M$ and $M subseteq L$.



          For $L subseteq M$, we will actually use structural induction, rather than induction on the length of the string. What this means is that we will induct on the fact that the string is in $L$ because it satisfies some part of the definition above, i.e it is built from strings we already know are in $L$; we then show that, assuming these strings are in $M$, so too is the string constructed from them.



          It is clear that $0, 01 in M$. So we have our base cases, and proceed with the inductive cases.



          Let $w = w_1 cdot w_2$ where $w_1, w_2 in L$. By induction hypothesis, this means that $w_1, w_2 in M$. It follows that neither of them contains a $11$ and both begin with $0$. Then $w$ also begins with $0$, and it does not contain a $11$ -- the only possible place this could happen is from the last character of $w_1$ and the first one of $w_2$, but $w_2$ begins with $0$. Thus, $w in M$.



          Now let $w cdot 0 = w'$ where $w' in L$. By induction hypothesis, $w' in M$, so it begins with $0$; so too must $w$. Additionally, $w$ cannot contain a $11$, as this would mean $w'$ does as well. Thus, $w in M$. (Actually we have to be a little careful here to consider the case where $w = epsilon$ separately, but you can do this case explicitly).



          This proves the inclusion $L subseteq M$. For the other direction, we proceed by strong induction on the length of the string. Base cases should be $epsilon, 0$; the inclusion of both of these in $L$ is straightforward.



          Let $w in M$ have length at least $2$. It must end in a $0$ or a $1$. If it ends in a $0$, then let $w = w' cdot 0$. Since $w in M$, it must begin with a $0$ and not contain $11$; then $w'$ does as well, so $w' in M$. But by induction hypothesis, this means $w' in L$, and $w = w' cdot 0$, both of which are in $L$; then $w in L$.



          If $w$ ends in a $1$, then we know that it must end in $01$ (it has length at least $2$ and cannot end in $11$). The same argument as before, taking $w = w' cdot 01$ suffices (this is why we need strong induction, or at least must consider all strings of length up to $n-2$ as well).






          share|cite|improve this answer









          $endgroup$



          I think it is most straightforward to prove both inclusions by induction. Namely, let $M$ be the language described by the condition you have above; we wish to show that $L subseteq M$ and $M subseteq L$.



          For $L subseteq M$, we will actually use structural induction, rather than induction on the length of the string. What this means is that we will induct on the fact that the string is in $L$ because it satisfies some part of the definition above, i.e it is built from strings we already know are in $L$; we then show that, assuming these strings are in $M$, so too is the string constructed from them.



          It is clear that $0, 01 in M$. So we have our base cases, and proceed with the inductive cases.



          Let $w = w_1 cdot w_2$ where $w_1, w_2 in L$. By induction hypothesis, this means that $w_1, w_2 in M$. It follows that neither of them contains a $11$ and both begin with $0$. Then $w$ also begins with $0$, and it does not contain a $11$ -- the only possible place this could happen is from the last character of $w_1$ and the first one of $w_2$, but $w_2$ begins with $0$. Thus, $w in M$.



          Now let $w cdot 0 = w'$ where $w' in L$. By induction hypothesis, $w' in M$, so it begins with $0$; so too must $w$. Additionally, $w$ cannot contain a $11$, as this would mean $w'$ does as well. Thus, $w in M$. (Actually we have to be a little careful here to consider the case where $w = epsilon$ separately, but you can do this case explicitly).



          This proves the inclusion $L subseteq M$. For the other direction, we proceed by strong induction on the length of the string. Base cases should be $epsilon, 0$; the inclusion of both of these in $L$ is straightforward.



          Let $w in M$ have length at least $2$. It must end in a $0$ or a $1$. If it ends in a $0$, then let $w = w' cdot 0$. Since $w in M$, it must begin with a $0$ and not contain $11$; then $w'$ does as well, so $w' in M$. But by induction hypothesis, this means $w' in L$, and $w = w' cdot 0$, both of which are in $L$; then $w in L$.



          If $w$ ends in a $1$, then we know that it must end in $01$ (it has length at least $2$ and cannot end in $11$). The same argument as before, taking $w = w' cdot 01$ suffices (this is why we need strong induction, or at least must consider all strings of length up to $n-2$ as well).







          share|cite|improve this answer












          share|cite|improve this answer



          share|cite|improve this answer










          answered Dec 10 '18 at 4:51









          plattyplatty

          3,370320




          3,370320












          • $begingroup$
            It is the first time I uses structrual induction. can you tell me how the hypothesis gonna be?
            $endgroup$
            – UltimateMath
            Dec 10 '18 at 8:47




















          • $begingroup$
            It is the first time I uses structrual induction. can you tell me how the hypothesis gonna be?
            $endgroup$
            – UltimateMath
            Dec 10 '18 at 8:47


















          $begingroup$
          It is the first time I uses structrual induction. can you tell me how the hypothesis gonna be?
          $endgroup$
          – UltimateMath
          Dec 10 '18 at 8:47






          $begingroup$
          It is the first time I uses structrual induction. can you tell me how the hypothesis gonna be?
          $endgroup$
          – UltimateMath
          Dec 10 '18 at 8:47




















          draft saved

          draft discarded




















































          Thanks for contributing an answer to Mathematics Stack Exchange!


          • Please be sure to answer the question. Provide details and share your research!

          But avoid



          • Asking for help, clarification, or responding to other answers.

          • Making statements based on opinion; back them up with references or personal experience.


          Use MathJax to format equations. MathJax reference.


          To learn more, see our tips on writing great answers.




          draft saved


          draft discarded














          StackExchange.ready(
          function () {
          StackExchange.openid.initPostLogin('.new-post-login', 'https%3a%2f%2fmath.stackexchange.com%2fquestions%2f3031736%2fregular-language-equality-prove-by-induction%23new-answer', 'question_page');
          }
          );

          Post as a guest















          Required, but never shown





















































          Required, but never shown














          Required, but never shown












          Required, but never shown







          Required, but never shown

































          Required, but never shown














          Required, but never shown












          Required, but never shown







          Required, but never shown







          Popular posts from this blog

          Le Mesnil-Réaume

          Ida-Boy-Ed-Garten

          web3.py web3.isConnected() returns false always